IMO 2014/2

Let n2n\geq2 be an integer. Consider an n×n n \times n chessboard consisting of n2n^2 unit squares. A configuration of nn rooks on this board is peaceful if every row and every column contains exactly one rook. Find the greatest positive integer kk such that, for each peaceful configuration of nn roots, there is a k×k k \times k square which does not contain a rook on any of its k2k^2 unit squares.

#Combinatorics #IMO

Note by Calvin Lin
6 years, 11 months ago

No vote yet
1 vote

  Easy Math Editor

This discussion board is a place to discuss our Daily Challenges and the math and science related to those challenges. Explanations are more than just a solution — they should explain the steps and thinking strategies that you used to obtain the solution. Comments should further the discussion of math and science.

When posting on Brilliant:

  • Use the emojis to react to an explanation, whether you're congratulating a job well done , or just really confused .
  • Ask specific questions about the challenge or the steps in somebody's explanation. Well-posed questions can add a lot to the discussion, but posting "I don't understand!" doesn't help anyone.
  • Try to contribute something new to the discussion, whether it is an extension, generalization or other idea related to the challenge.
  • Stay on topic — we're all here to learn more about math and science, not to hear about your favorite get-rich-quick scheme or current world events.

MarkdownAppears as
*italics* or _italics_ italics
**bold** or __bold__ bold

- bulleted
- list

  • bulleted
  • list

1. numbered
2. list

  1. numbered
  2. list
Note: you must add a full line of space before and after lists for them to show up correctly
paragraph 1

paragraph 2

paragraph 1

paragraph 2

[example link](https://brilliant.org)example link
> This is a quote
This is a quote
    # I indented these lines
    # 4 spaces, and now they show
    # up as a code block.

    print "hello world"
# I indented these lines
# 4 spaces, and now they show
# up as a code block.

print "hello world"
MathAppears as
Remember to wrap math in \( ... \) or \[ ... \] to ensure proper formatting.
2 \times 3 2×3 2 \times 3
2^{34} 234 2^{34}
a_{i-1} ai1 a_{i-1}
\frac{2}{3} 23 \frac{2}{3}
\sqrt{2} 2 \sqrt{2}
\sum_{i=1}^3 i=13 \sum_{i=1}^3
\sin \theta sinθ \sin \theta
\boxed{123} 123 \boxed{123}

Comments

The answer is n1.\left \lfloor \sqrt{n-1} \right \rfloor. Rather simple for an IMO P2; I suppose it's a C2 in the shortlist. The main idea is to tile the grid into several disjoint j2×j2j^2 \times j^2 tiles where j=n1.j= \left \lfloor \sqrt{n-1} \right \rfloor. I have posted my solution in AoPS, I can post it here if necessary. :)

Sreejato Bhattacharya - 6 years, 11 months ago

Log in to reply

Could you please post the full solution for this problem?

Saksham Bansal - 6 years, 11 months ago

I didn't like the phrasing of this question because it was slightly ambiguous.
- It wasn't clear to me that kk should depend on nn. Would have preferred if they called it knk_n instead.
- It wasn't clear if the square have to be consecutive rows and columns, or could be a sub-matrix (i.e. you get to pick kk rows and kk columns). I'm guessing the answer is "have to be consecutive".

Calvin Lin Staff - 6 years, 11 months ago

Log in to reply

Since k×kk\times k is a square, it has to be consecutive.

Kenny Lau - 6 years, 11 months ago

Conjecture: k=n2k=\left\lfloor\frac n2\right\rfloor

Kenny Lau - 6 years, 11 months ago

Log in to reply

xxxx \begin{array} {| l | l | l | l |} \hline x & & & \\ \hline & & x & \\ \hline & x & & \\ \hline & & & x \\ \hline \end{array}

seems to be a counterexample to n=4n=4. There is no 2×2 2 \times 2 (consecutive) square.

Calvin Lin Staff - 6 years, 11 months ago

Log in to reply

My mistake, I thought we're finding the maximum value of kk.

Kenny Lau - 6 years, 11 months ago

Log in to reply

@Kenny Lau It is "for each peaceful configuration of nn roots". I've noticed that you made a similar interpretation error in another problem, which asked for the "smallest needed", which could be interpreted as "the most ever".

Otherwise, your solution works for the problem that you're solving.

Calvin Lin Staff - 6 years, 11 months ago

If k>n2k>\left\lfloor\frac n2\right\rfloor, there will be (nk)(n-k) rows and (nk)(n-k) columns left, which is not enough to put all rooks, because 2(nk)=2n2k<n2(n-k)=2n-2k<n.

Kenny Lau - 6 years, 11 months ago
×

Problem Loading...

Note Loading...

Set Loading...